Using Remainder Theorem to find remainder

In summary: Thanks for your help!In summary, the problem asks for a quotient and remainder for a situation where 3y - √(2) = 0. The attempt at a solution is to use synthetic division to find a quotient and remainder for y = √(2)/3. The remainder is -21 when the process ends.
  • #1
Schaus
118
5

Homework Statement


(y4 - 5y2 + 2y - 15) / (3y - √(2))
The answer says (2√(2)/3)-(1301/81)
https://lh3.googleusercontent.com/tdxTK9jIztkS5_7q9x0jP_D3HdkcDcKO9K_T2glYcdlF5MW71SiVq0xNjj09bkX3yUTi0lVS5aehQcTCpjVMgEyIEySNwanKwnxkfuh-l9wGnecvNy11IkibZd5nNgn4eiv4l3I-SGY89O9V_M2uzlcRBLfMVMkS_gybhaz3be3-eDmspF47-z1thzMs6iseX2eU2IjJMpAmx0YjN4mrOY4Bx7hQtrUnyMA4GSXyavCjbLx89CaKmAU6bbqm36Qqm3ieAW1299QOvcXrBqH2XaT_u9T_THz1ktwI9a_Mw5ozQurH0RnZvOIWbArej-6PU9mOWIpvasISRaLkwCWpG_w76wjFjZGISRXhq33LvWxM4bGFbvirN6c--KFp5yLWC7djePdwbpb69UF5FbPYJxOuBIWoaVg1QPwwg0V7im1Zp0fThljuNWHmwyM1vA6tXhnc5zkx7WnZ8BCiP2feBXWOdal8yvnTA0BXY8O0_Ly1YSO8fqX1mATZ8RY8YlawtMB_FMwuNsAYVJ6Vde0tdPFQEYlQ3yYERXpeo-31kNJRs_Y9wP5pWUp58FTVgyBsOmwiztqNVUVJyQfnMiKjDKJeR9_wevbfnEStla1ntJRTakXqP8dG=w449-h311-no

Homework Equations

The Attempt at a Solution


Using synthetic division
3y - √(2) = 0
3y = √(2)
y = √(2)/3
View attachment 111547

My final answer that I keep getting is (2√(2)/3)-21. I can't seem to get (2√(2)/3)-(1301/81). I was just wondering if someone could show me where I'm going wrong? Sorry writing this out was very difficult so I hope you can still understand it.
 
Last edited:
Physics news on Phys.org
  • #2
I don't actually can follow your calculations. If I compare my calculation with your result, then I think you simply lost some denominators.
I get ##\frac{4}{81}-\frac{10}{9}-15+\frac{2\sqrt{2}}{3}## whereas you seem to have added only the nominators ##4-10-15+\frac{2\sqrt{2}}{3}##.

I once gave an example on PF here:
https://www.physicsforums.com/threa...r-a-polynomial-over-z-z3.889140/#post-5595083

Maybe it helps.
 
  • Like
Likes Schaus
  • #3
upload_2017-1-13_19-24-50.png
Sorry, I should have just posted this to begin with.
 
  • #4
Schaus said:
View attachment 111547 Sorry, I should have just posted this to begin with.

Your long-division process is all wrong. Let ##p(y) = y^4 - 5y^2 + 2y - 15## and ##d(y) = 3y - \sqrt{2}##. You want to find a "quotient" ##q(y)## and a number ##r## (the "remainder") that give you ##p(y) = q(y) d(y) + r.## The question is asking you to find ##r##.

Step 1: see how many times the leading term of ##d(y)## will go into the leading term of ##p(y)##; that is the number of times ##3y## goes into ##y^4##. The answer in this case is ##y^4/(3y) = (1/3)y^3##, so that is the first term in your quotient ##q(y)##. Now we have
$$ p(y) - \frac{1}{3} y^3 d(y) = \frac{\sqrt{2}}{3} y^3 -5 y^2 + 2y - 15 \equiv p_1(y).$$

However, all that would be doing it the hard way. The easy way would be to go ahead and use the "remainder theorem".

Step 2: see how many times the leading term of ##d(y)## goes into the leading term of ##p_1(y)##. The answer is ##(\sqrt{2}/3)/(3 y) = (\sqrt{2}/9) y^2##, so that is the next term in your quotient ##q(y)##. We have
$$p_2(y) \equiv p_1(y) - \frac{\sqrt{2}}{9} y^2 d(y) = -\frac{43}{9} y^2 + 2y - 15.$$

So, up to now we have
$$p(y) = \left( \frac{1}{3} y^3 + \frac{\sqrt{2}}{9} y^2 \right) d(y) + p_2(y).$$

Keep going like that; the next term in the quotient ##q(y)## will be the number of times the leading term of ##d(y)## goes into the leading term of ##p_2(y)##, so is the number of times ##3y## goes into ##-(43/9) y^2##, etc., etc.

The remainder will be what you have left when the process comes to an end.

However, all that would be doing it the hard way; I included the material just because you attempted to do it, but did it all wrong. The easy way would be to just use the so-called "remainder theorem".
 
  • Like
Likes Schaus
  • #5
I was using synthetic division which worked on every other question but I'll try it the way you're suggesting it.
 
  • #6
I got the same result as the answer. Thanks for your help!
 
  • #7
Schaus said:
I was using synthetic division which worked on every other question but I'll try it the way you're suggesting it.

It looks to me like you simply made a couple multiplication errors when working with fractions. I worked the problem with synthetic division and got the right answer.
 
Last edited:

Related to Using Remainder Theorem to find remainder

1. What is the Remainder Theorem?

The Remainder Theorem is a mathematical concept that states that when a polynomial P(x) is divided by a binomial (x-a), the remainder will be equal to P(a).

2. How is the Remainder Theorem used to find the remainder?

To find the remainder when a polynomial P(x) is divided by (x-a), simply plug in the value of a into the polynomial and solve for the remainder. This is because the remainder is equal to P(a) according to the Remainder Theorem.

3. Can the Remainder Theorem be used for any type of polynomial?

Yes, the Remainder Theorem can be used for any type of polynomial, including monomials, binomials, and polynomials with multiple terms.

4. What is the purpose of finding the remainder using the Remainder Theorem?

The main purpose of using the Remainder Theorem is to simplify division of polynomials and to determine factors of a polynomial. It can also be used to check the accuracy of long division.

5. Are there any limitations to using the Remainder Theorem?

One limitation of the Remainder Theorem is that it can only be used when dividing by a binomial (x-a). Additionally, it cannot be used to find the quotient of the division, only the remainder.

Similar threads

  • Precalculus Mathematics Homework Help
Replies
7
Views
678
  • Precalculus Mathematics Homework Help
Replies
4
Views
862
  • Precalculus Mathematics Homework Help
Replies
15
Views
2K
  • Precalculus Mathematics Homework Help
Replies
4
Views
1K
  • Precalculus Mathematics Homework Help
Replies
5
Views
2K
  • Precalculus Mathematics Homework Help
Replies
9
Views
2K
  • Precalculus Mathematics Homework Help
Replies
33
Views
3K
  • Precalculus Mathematics Homework Help
Replies
3
Views
3K
  • Precalculus Mathematics Homework Help
Replies
8
Views
2K
Replies
4
Views
3K
Back
Top